Sie sind auf Seite 1von 110

1

Chapter

I
F orce System

-any arrangement where two or more forces act on a body or on a group of related
bodies.

F1 F2 F2
F1
A A
B C
F3
F3
2

Coplanar
- the lines of action of all the forces in a force system lie in one plane.
Non-coplanar
- the lines of action lie in three dimensions.

F2 F1

F1
F2

Z
3

F3

Chapter

II
R esultant

- the effect of a system of forces on a body.

F F2
R
Fy
𝜃 𝜃
Fx F1

Fx free vector

Fy
Where: free vector
Fy
F= √Fy 2 + Fx 2 Fx
Fy
Tanθ=
Fx
4

Problem: A force of 200lb is directed as shown below. Determine the x and y


components of the force.
200

Fy
30°

Fx
Fx = F cos θ
Fx = 200 cos 30°
Fx = −173.21lb
Fx = 173.21lb ←

Fy = F sin θ
Fy = 200sin30°
Fy = +100lb
Fy = 100lb ↑
5

Problem: Determine the resultant and its component forces along x and y-axis.
Fy

A = 390lb

67.38°
Fx
30°
40°
300lb
B = 400lb

Force θ x (Fcosθ) y (Fsinθ)

A 390 67.38° −150 360

B 400 40° −306.42 −257.12

C 300 30° 259.81 −150

R x = −196.61lb R y = −47.12

R x = 196.61lb ←
R y = 47.12lb ↓

R = √Rx 2 + Ry 2

R = √(196.61)2 + (47.12)2

R = 202.18lb

Ry
tanθ =
Rx
47.12
θ = tan−1 ( )
196.61
θ = 13.29°
6

Problem: Compute R, Rx, Ry, and θ.

90°
C = 100lb
70°

30°
Fx

Fy

Magnitude θ x (F cos θ) y (F sin θ)


Forces
A 60 30° 51.96lb 30lb

B 90 40° −68.94lb 57.85lb

C 100 50° −64.28lb −76.60

Rx=−81.26lb Ry=11.25lb

R = √(11.25)2 + (81.26)2

R = 82.04lb
Ry
tan θ =
Rx
11.25
θ = tan−1 ( )
81.26
θ = 7.88°
7

Problem: Two jeeps on opposite banks of the canal pull a banca moving parallel the
riverbanks by means of two cables. The tensions in the cables are 200kN and
240kN, while the angle between them is 60°. Find the resultant pull on the canal and
the angle between each of the cable and side of the canal.
200KN

60°
𝛽

240 KN

α = 60° − β
∑Fy = R y ↑ +

Ry=0- 200sinα − 240sinβ


But sinα = sin(60 − β)
= sin60cosβ − sinβcos60
0 = 200(sin60°cosβ − sinβcso60°) − 240sinβ
0 = 200sin60°cosβ − 200sinβcso60° − 240sinβ
0 = 173.21cosβ − 100sinβ − 240sinβ
340sinβ = 173.21cosβ
sinβ 173.21
=
cosβ 340
173.21
tanβ = 340

β = 27°α = 33°

∑Fx = R x → +
Rx=-200cos33°-240cos27°
Rx = 381.58KN ←
8

Moment of Force

-the ability of the force to produce turning or twisting moment about an axis.

Problem: Determine the position and magnitude of the resultant.

30KN 60KN 40KN

20KN

2m 2m 2m 4m
R = −30 − 60 − 40 + 20
R = −110kN
R = 110KN ↓

∑Ma = R x ↻ +
R x = 30x30 + 60x60 + 40x40 − 20x20
; x30 = 2m ; x60 = 5m ; x40 = 11m ; x20 = 7m
110x = 30(2) + 60(5) + 40(11) − 20(7)
x = 6m
9

Problem: The beam AB in the figure supports a load which varies from an intensity
kN kN
of 50 m to 200 m . Calculate the magnitude and position of the resultant load.

200KN/m

50 KN/m

L = 12m

Rectangular load Triangular load

Rr
Rt

R R = WL = 50(12) = 600KN
WL 50(12)
RT = = = 300KN
2 2

R = −R R − R T
R = −600 − 300
R = −900KN

∑ MA = R x ↻ +

900x = 600(6) + 300(8)


x = 6.67m
R = 900KN ↓
10

Problem: The three step pulley shown in the figure is subjected to the given couples.
Compute the value of the resultant couple. Also, determine the forces acting at the
rim of the middle pulley that are required to balance the given system.

40KN

30KN

60KN

16m
8m
12m
60KN

30KN

40KN

∑Mx = R x ↻ +
R x = −60(8) + 30(12) − 40(16)
R x = −760KN − m

R x = Fd
760 = F(12)
F= 63.33KN (couple)
11

Problem: Locate the magnitude and position of the resultant force of the loads acting
on the fink truss shown below. 400KN

300KN 300KN

200KN 200KN

30° 30°

10m 10m 10m 10m

Compute for z
5
= tan 30°
z
5
z = tan 30°

z = 8.66m

v
cos 30° =
z
v = 7.50m

∑Fy = R y ↑ + ; R=R y

R = −200 − 300 − 400 − 300 − 200


R = −1400KN
R = 1400KN ↓

∑MA = R x ↻ +
1400(x) = 200(0) + 300(7.50) + 400(20) + 300(40 − 7.50) + 200(40)
x = 20m
12

Problem: The howe roof truss shown below carried the given loads. The wind loads
are perpendicular to the inclined members. Determine the magnitude of the resultant
its inclination with the horizontal and where it intersects AB.
1120KN

2240KN

10 m 2000KN
1120KN
5m 5m θ
θ
10m 10m 10m 10m

3,000KN 2,000KN 1,000KN

β = 90 − θ
β = 90 − 25.97
β = 63.43°

5
tan θ =
10
θ = 26.57°

4480y= 4480sin 63.43°


4480y= 4006.84KN
4480x= 4480cos 63.43°
4480x= 2003.86KN

∑Fx = R x → +
R x = 2000 + 2003.86
R x = 4003.86KN →
∑Fy = R y ↑ +
13

R y = −4006.84 − 3000 − 2000 − 1000

R y = 10006.84KN ↓

R = √R x 2 + R y 2

R = √(4003.86)2 + (10006.84)2
R= 10778.11KN
14

Resultant

Problem: Determine and locate the resultant R of the two forces and one couple
acting on the I-beam.
8KN

x 25 KN - m

2m 2m 2m
5KN
Solution
∑Fy = 0 ↑ +

R = 8−5 = 3KN ↓

∑MO = 0 +

0 = 25−5(2)−8(2) = 1KN-m

3d = 1
d = 1/3m
x = 1.333m
15

Problem: If the resultant of the two forces and couple M passes through point O,
determine M.

150mm

M 150mm 30°
O 320N

400N
160mm

∑MO = 0 +
0= M –(400)(0.15cos30)−(320)(0.3)
M = 148 N-m
16

Problem: The directions of the two thrust vectors of an experimental aircraft can be
independently changed from the conventional forward direction within limits. For the
thrust configuration shown, determine the equivalent force-couple system at point O.
Then replace this force-couple system by a single force and specify the point on the
x-axis through which the line of action of this resultant passes.

10m

T A

3m O
x
3m

T B

15 N

Solution:
The force-couple equivalent system at point O is first determined as
followed.
R = (T + T cos15)i + (T sin15) j = 1.966Ti + 0.259Tj N

∑MO = 0 +
0 = (T cos15)(3)−T (3)−(T sin15)(10) = 2.69T N-m
17

Problem: Calculate the moment of the 1200N force about pin A of the bracket. Begin
by replacing the 1200N force by a force-couple system at point C. Calculate the
moment of the 1200N force about the pin at B.

2 600 mm
1

C
A

200 mm 500 mm
1200 N
B

Solution:
Moment of 1200 N force about C is:
MC = 1200×0.2 = 240 N-m

Therefore the moment about point A and B are:


MA = MC+(1200) (1 √5 )(0.6)= 562 N-m

MB = MA+(1200)(2 √5 )(0.5) = 1099 N-m


18

Problem: Determine completely the resultant of the concurrent force system shown
in this figure.

Y
300 lb

200 lb

60° 30°
400 lb X
45°

100 lb

50 lb

∑FX = 0 → +
= 200 cos 30° + 100 cos 45° - 400 cos 0° - 300 cos 60°
= 173.2 + 70.7 – 400 – 150
∑FX = −306. 1lb

∑FY = 0 ↑ +
= 200 sin 30° + 300 sin 60° - 50 sin 90° - 100 sin 45°
= 100 + 259. 8 – 50 – 70.7
∑FY = +239. 1lb

R = √306.12 + 239.12 = 388lb


239.1
θ = tan−1 (306.1) = 38°
19

Chapter

III
E Equilibrium of Forces

- it is the term used to designate the condition where the resultant of a


system of force is zero.

Free Body Diagram (FBD)


- is isolated view of the body that shows the external forces exerted on the body.
External Force
– caused either by direct bodily contact or by gravitational or magnitude attraction.
20

Problem: The cable shown in the figure below support load of 600KN. Determine the
tensile force T in the cable and the compressive force C in the boom.

30°

45°

600KN

∑Fy = 0 ↑ +

0=Tw−600
Tw= 600kN
∑Fx = 0 → +

0=−Tx + Cx ①

Tx = T cos 30° ②

Cx = C cos 45° ③

②, ③ in ①
0 = −T cos 30° + C cos 45°
C cos 45°
T= ④
cos 30°

∑Fy = 0 ↑ +

0 = Ty + Cy − Tw

0 = T sin 30° + C sin 45° − 600


cos 45°
0 = C( ) sin 30° + C sin 45° − 600
cos 30°
C = 538.12kN Compression
T = 439.37 Tension
21

Problem: The 300KN sphere is supported by the pull P and a 200KN weight passing
over a frictionless pulley. If α = 30°, compute the values of P and ϕ.

𝛼 = 30°
𝜃 P

200KN

∑Fy = 0 ↑ +

P cos ϕ + 200 cos 30° = 300


200 sin 30°
P= eq. 1
sin ϕ

∑Fx = 0 → +
0=-300+200cos30°+Pcos∅ eq. 2

Eq.1 @ Eq.2
200 sin 30°
( ) cos ϕ + 200 cos 30° = 300
sin ϕ
(200 sin 30°) cot ϕ + 200 cos 30° = 300
100 cot ϕ + 173.21 = 300
126.79
cot ϕ =
100
ϕ = 38.26° eq. 3
22

Eq.3 @ Eq. 1
P sin ϕ = 200 sin 30°
200 sin 30°
P=
sin 38.26°
P = 161.49KN
23

Problem: Three bars hinged at A and D and pinned at B and C as shown. Form a
four-link mechanism. Determine the value of P that will prevent motion

200 KN C

B 60°
45°
45°

75°

D
A

@ point B

∑Fy = 0 ↑ +

AB sin 30° = 200 cos 45°


AB = 163.3KN

∑Fx = 0 → +
BC = AB sin 30° + 200 sin 45°
BC = 223KN

@ point C
∑Fy = 0 ↑ +

P cos 45° = BC cos 15°


P cos 45° = 223 cos 15°
P = 304.6KN
24

EQUILIBRIUM

Problem: Three cables are joined at the junction ring C. Determined the tensions of
the cables AC and BC caused by weigh of the 30kg cylinder.

A D
45° TAc 30(9.81)
C
15°
45° 15°
60°
30° 30 kg
TBc
B

∑Fy = 0 ↑ +

Tac sin45° −TBc sin60°+30(9.81)sin15° =0


∑Fx = 0 → +
-Tac cos45° −TBc cos60°+30(9.81)cos15° =0

Solve simultaneously to obtain


TAc = 215N
TBc = 264N
25

Problem: The 600lb drum is being hoisted by the lifting device with hooks over the
end lips of drum. Determined the tension T in each of the equal length rods which
form the two U- shaped members of the device.

600lb
10”

𝜃 𝜃

36”

10
θ =tan−1 (32/2)

θ =29.1°

∑Fy = 0 ↑ +

600- 4Tsin24.1° =0
T= 309lb
26

Problem: Determined the external reactions at A and F for the roof truss located as
shown. The vertical loads represent the effect of the supported roofing materials,
while the 400-N force represents a wind load.
500N

C 500N

400N 60°
B D 500N

250 N E 250N

A 60° 30° 30° 30° F x

10m

500N

C 500N
60°
400N D 500N
B
E 250N
250 N
F
Ax A 60° 30° 30°
30°
2.5 m 2.5 m 2.5 m 2.5 m
Ay Fy
∑Fx = 0 → +
-Ax+400cos30°=0
Ax = 346N

∑MA = 0 +
10
400( 4 )+500(2.5) +500(5) + 500(7.5) + 250(10) – 10 Fy=0

Fy = 1100N

∑Fy = 0 ↑ +

250 - 400sin30° - 500(3) – 250 + 1100 + Ay = 0


Ay= 1100N
27

Problem: To accommodate the rise and falls of the tide, a walkway from a pier to a
float is supported by two rollers as shown. If the mas – center of 300kg walkway is at
G, calculate the tension T in the horizontal cable which is attached to the cleat and
find the forces under the roller at A.

4m B T
4m
G
A

T
4m

y 4m

30°

∑MA = 0 +
300(9.81) 4cos330°-8 Acos30° = 0
A = 1472N

∑Fy = 0 ↑ +

Bcos30°+ 1472 – 300 cos 30°(9.81) = 0


B = 1699N

∑Fx = 0 → +
T – 1699sin30°=0
T = 850N
28

Problem: What Horizontal force P must a worker exert on the rope to position the
50kg crate directly over the trailer?

4m 𝜃
4m R R

P 𝜃
1m
2m

50 (9.81)

P = 50 (9.81) tan θ
1
Sinθ= 4 = 0. 25

or

Tanθ = 1 / √42 − 12 = 0.258


P = 50 (9.81) (tanθ) = 126 .6
29

CHAPTER

IV
A
1.Joint Method
NALYSIS OF STRUCTURE

2. Section Method
30

Problem: Find the internal load of member IC.

10KN 20 KN
J

5KN
I K

H 9m
L
6m
A 3m θ G
5m B 5m C 5m D 5m E 5m5m F

3m

5m
3
tanθ = 5 = 30.96°

Joint Method

20x= 20sin 30.96°


20x= 10.29KN

20y= 20cos 30.96°


20y= 17.15KN
31

∑Fho = 0 → +
0=RAh-10.29
Rah= 10.29KN

∑Fv = 0 ↑ +
0=-5-10-17.15+RAv+RGv Eq.1

∑MA = 0 +
0= -10.29(6) + 5(5) + 10(10) +17.15(20) – RGv (30)
RGv=13.54KN ↑

RGv in Eq.1
0= -5-10 -17.15+ RAv+13.54
RAv = 18.61KN ↑

FBDa

AH

θ
AB

∑Fv = 0 ↑ +
0= AHsin30.96+18.61
AH=-36.18KN (C)
32

∑Fh = 0 → +
0=10.29-36.18cos30.96 + AB
AB=-20.74KN
AB=20.74KN (T)

FBDb HB

20.74 BC

∑Fv = 0 ↑ +
0=HB

∑Fh = 0 → +
0= -20.74+ BC
BC=20.74KN (T)

5KN
`

AH θ
20°
HC

∑Fv = 0 ↑ +
0=-5c0s30.96 + HCsin20 (30.96)
HC = 4.86KN (C)
33

HC IC

BC 𝜃 CD

∑Fv = 0 ↑ +
0= IC-HCsin30.96
IC = 2.50KN (T)

Section Method
10KN
5KN

RAh C

Slice
RAv

RAh

RAv

∑MA = 0 +
0= - IC (10) + 5(5)
IC= 2.5KN
34

ANALYSIS OF STRUCTURES
Problem:Calculate the force in each member of loaded truss.

E D 2KN

3m

A
C
3m 3m
B
∑MA = 0 +
6 Ay – 2 (3) = 0
Ay = 1KN

Cx = 2KN
Cy = 1KN

Joint A
AE
∑Fy = 0 ↑ +

1 – AEsin45° = 0
45°
AE = 1.414KN (C) AB

∑Fx = 0 → +
AB – 1.414cos45° =0
1KN
AB = 1KN
35

Joint E

∑Fx = 0 → +
1.414sin45° - DE =0
DE
DE = 1 KN (C)
45°

∑Fy = 0 ↑ +

1.414cos45° - BE = 0
BE
BE = 1KN (T) 1.4 14 KN

Joint B
1KN BD
∑Fy = 0 ↑ +

1 – BDsin45° = 0
BD = 1.414KN(C)
1KN BC
∑Fx = 0 → +
BC – 1.414cos45° − 1=0
BC = 2KN (T)
CD
Joint C

∑Fy = 0 ↑ +

CD – 1 = 0 2KN 2KN
CD = 1KN (T)

Joint D checks 1KN


36

Problem: Calculate the forces in members CF, CG, and EF of the loaded truss.

2000lb
E D

10’
F C 2000lb

10’
G B

10’
H A

26’

Joint E :
DE = EF = 0

Joint D

2000 lb

DF
CD
37

Joint F
2520lb
−1 3
( α = tan ( ) = 16.70°)
10
θ
∑Fy = 0 ↑ +
CF
2520 cosθ – FG cosα = 0
FG = 1606lb (C)
α
FG
∑Fx = 0 → +
2520 sinθ – CF - 1606 sinα = 0
CF = 1538lb (C) 1538lb
Joint C
16
(β = tan−1 (10) = 58.0°)
1538lb 2000lb
∑𝐅𝐱 = 𝟎 → +
1538 + 2000 - CG sinβ = 0
CG
CG = 4170lb (T) CB
38

Problem: The movable gantry is use to erect and prepare a 500- MG rocket for
firing. The primary structure of the gantry is approximated by the symmetrical plane
truss shown, which is indeterminate. As the gantry is positioning a 60 – MG section
of the rocket is suspended from A, strain-gage measurement indicates a
compressive force of 50 KN in member AB and a tensile force of 120 KN in member
CD due to the 60 -

H C D
16m

J A B E

G F

7 panels at 12m

16m 24m 16m

By symmetry, AJ = AB , CH = CD , BC = JC
Joint C y
∑Fy = 0 ↑ +
4 120KN 3 120KN
(2 BC) – 60 (9.81) = 0
5
4
BC = 368KN (C)

60 (9.81)
39

Joint D

∑Fx = 0 → +
DE 120KN
– 120 = 0
√2
45°
DE = 169.7KN (T)
DE
BD
∑Fy = 0 ↑ +

BD = 120KN (C)

Joint E
AE = 1.414 KN - C
∑Fx = 0 → +
∑𝑭𝒙 = 𝟎 → + 45°
BE = 120KN (C) BE
AB – 1.414cos45°=0
AB = 1 KN FE
∑Fy = 0 ↑ +

EF = 120KN (T)

Joint B
368KN 120KN
∑Fx = 0 → +
4 3
BF + 120 - 5 (368) = 0 120KN
5

BF = 188.4KN (C) BF
BC
40

Problem: Determine the force in member BC, CG, and GF of the Warren truss.
Indicate if the members are in tension or compression.

B 3m C 3m D

3m 3m

A E
3m G 3m F 3m

6KN 6KN

Support Reaction
B FBC C
+ ∑ME = 0
6(6) + 8(3) – Ay (9) = 0
3m
Ay = 6.667KN 3 sin60°

∑Fx = 0 → +
Ax = 0 3m
G
FGF

Ay = 6.667 6KN
Section Method
+∑MC = 0
FGF(3 sin 60°)+ 6 (1.5) – 6.667 (4.5) = 0
FGF = 8.08KN (T)

+ ∑MG = 0
FBC ( 3 sin 60°)– 6.667(3) = 0
FBC = 7.70KN (C)
41

↑+∑Fy = 0
6.667 – 6 -FCG sin 60° = 0
FCG = 0.77KN (T)
42

CHAPTER

V
F ORCE SYSTEM IN SPACE

F2=Fx2+Fy2+Fz2
Fy
Fx=Fcos θxor cos θy = F
Fy
Fy= Fcos θy or cos θy = F

Fz
Fz= F cos θz or cos θz = F

Fx Fy Fz F
= = =
x y z d

also;
d2 = x 2 + y 2 + z 2
x = |x2 − x1 |

𝑦 = ⃒𝑦2 − 𝑦1 ⃒
z = |z2 − z1 |

F = √Fx 2 + Fy 2 + Fz 2
43

Problem: The resultant of the concurrent forces has a magnitude 0f 100KN and acts
through the origin and points.
x=2, y= 3, z=4

d = √22 + 32 + 42
d = 5.39m

Fx 1000
=
2 5.39

Fx = 371.06KN

Fy 1000
=
3 5.39

Fy = 556.59KN

Fz = 742.12KN

Fx 371.06
cos θx = =
F 1000
θx = 68.22°
44

Fy 556.59
cos θy = =
F 1000
θy = 56.18°

Fz 1000
=
4 5.39

Fz 742.12
cos θz = =
F 1000
θz = 42.07°
45

Problem: The unsymmetrical framework below supports a vertical load of 1,700lb at


point/joint A. Points C and B are in the same vertical plane while B is 3ft in front of
this plane. Compute the force in each member.

D
2’

4’
8’
E 90°
C
8’

6’

1700 lb
4’
3’ B

dAB = √(x2 − x1 )2 + (y2 − y1 )2 + (z2 − z1 )2

dAB = √(−5 − 0)2 + (−6 − 0)2 + (4 − 0)2


dAB = 8.77ft

dAC = √(x2 − x1 )2 + (y2 − y1 )2 + (z2 − z1 )2

dAC = √(−8 − 0)2 + (4 − 0)2 + (8 − 0)2


dAC = 12ft

dAD = √(x2 − x1 )2 + (y2 − y1 )2 + (z2 − z1 )2

dAD = √(−8 − 0)2 + (4 − 0)2 + (−2 − 0)2


dAD = 9.17ft
46

by matrix
AD
|AB| = (Mat A × Mat B −1 )−1 × Mat C
AC

AD AB AC
−8 −5 −8 x
Mat A (coordinates) = | 4 −6 4 |y
−2 4 8 z

9.17 0 0
Mat B (distance) = | 0 8.77 0 |
0 0 12

0
Mat C (load) = |−1700|
0

−183.4 AD
Answer= | 1754 | AB
−1260 AC

AB = 1754lb (compression)
AC = 1260lb (tension)
AD = 183.4lb (tension)

Solution
∑Mz = 0 ↻ +

1700(8) − Bx (10) − By (3) = 0 ①②③④


Bx By
=
x y
Bx By
=
5 6
47

6
By = Bx ②
5

② in ①
6
1700(8) − Bx (10) − Bx (3) = 0
5
Bx = 1000

AB = B
B Bx
=
dAB x
B 1000
=
8.77 5
B = 1754lb

∑My = 0 ↻ +

Cx (8) − Dx (2) − Bx (4) − Bz (3) = 0


Cx (8) − Dx (2) − 1000(4) − 800(3) = 0
8Cx − 2Dx − 4000 − 2400 = 0

8Cx − 2Dx − 6400 = 0③

∑Fx = 0 → +
Cx − Dx + 1000 = 0
Cx + Dx = 1000

Cx = 1000 − Dx ④

④ in ③
8(1000 − Dx ) − 2Dx − 6400 = 0
Dx = 160
48

D Dx
=
dAD x
D 160
=
9.17 8
D = AD = 183.4lb

Dx in ④
Cx = 1000 − 160
Cx = 840
C Cx
=
dAC x
C 840
=
12 8
C = AC = 1260lb
49

Force System
Problem: The framework shown consists of three members AB, AC and AD whose
lower ends are in the same horizontal plane. A horizontal force of 1000 N is acting
parallel to the x-axis is applied at A.

Y
1000 N
A

6m

X C
3m B
3m 3m

5m
D
Z

① Compute the force in member AB.

② Compute the force in member AC.

③ Compute the force in member AD.


50

① Force in member AB

AB = √(3)2 + (6)2
AB = 6.71

AC = √(3)2 + (3)2 + (6)2

AC = 7.35

AD = √(5)2 + (3)2 + (6)2

AD = 8.37

∑ My = 0

3Cx = 5Dx
Bx + Cx + Dx = 1000

∑ Mz = 0

6By = 1000(6)

By = 1000

By FAB
=
6 6.71
1000 FAB
=
6 6.71
FAB = 1118 N (Compression)

② Force in member AC
Bx FAB
=
9 6.71
1118(9)
Bx =
6.71
51

Bx = 1500 N

∑ Mx = 0

3Cy = 5Dy

Cy + Dy = By

Cy + Dy = 1000
3Cy
Cy + = 1000
5
5000
Cy = = 625
8

Cy FAC
=
6 7.35
625(7.35)
FAC =
6
FAC = 766 N (tension)

③ Force in member AD
Dy + 625 = 1000

Dy = 375

Dy FAD
=
6 8.37
375 FAD
=
6 8.37
FAD = 523 N (tension)
52

Problem: The points B, C and D of the cantilever framework shown are attached to a
vertical wall. The 400N load is parallel to the z-axis and the 1200N load is vertical.

6m
B
4m 400N

0 10m A
C 4m x
z 1200N

① Compute the force in member AB.

② Compute the force in member AC.

③ Compute the force in member AD.

Solution:

① Force in member AB

∑ Mz = 0

1200(10) = Dx (6)
Dx = 2000

AB = √(4)2 + (50)2

AB = 10.77
53

AC = √(4)2 + (10)2

AC = 10.77

AD = √(6)2 + (10)2

AD = 11.66

∑ Fh = 0

Dx = Bx + Cx
Bx + Cx = 2000

∑ My = 0

4Cx + 400(10) = Bx (4)


CX + 1000 = Bx
Bx − Cx = 1000

Bx − Cx = 1000
Bx + Cx = 2000
2Bx = 3000
Bx = 1500

Cx = 1500 − 1000
Cx = 500

∑ Mx = 0

4Cy = 4By + Dz (6)


54

Bx FAB
=
10 10.77
1500 FAB
=
10 10.77
FAB = 1615.5N
55

Problem: The framework shown supports a vertical load of 2000KN points B, C and
D are in the same horizontal plane. Determine the force in each member.

2000 N

10m
z
C
B 4m
4m

8m
x

Solution: 6m
D
∑ Mz = 0
8Dy − 4By = 0

By = 2Dy

∑ Mx = 0

4Cy − 6Dy = 0
3Dy
Cy =
2

∑ Fy = 0

By + Cy + Dy = 2000
56

3Dy
2Dy + + Dy = 2000
2
Dy = 444.44KN

d1 = √(10)2 + (8)2 + (6)2 = 14.14

AD Dy
=
d1 y
AD 444.44
=
14.14 10
AD = 628.44 kN (Compression)

3Dy 3(444.44)
Cy = = = 666.66 kN
2 2
d2 = √(4)2 + (10)2 = 10.77

AC Cy
=
d2 y
AC 666.66
=
10.77 10
AC = 717.99 kN (Compression)

By = 2Dy = 2(444.44) = 888.88 kN

d3 = d2 = 10.77

AB By
=
d3 y
AB 888.88
=
10.77 10
AB = 957.32KN
57

Problem: The shear-leg derrick shown in the figure supports a vertical load of 2000
N applied at A. points B, C, and D are in the same horizontal plane and A, O and D
are in the xy plane.

20m y 10m
A

15m
z
2000 N

D B
0 5m x
10m

① Compute the force in member AB.

② Compute the force in member AC.

③ Compute the force in member AD.

Solution:

① Force of member AB

∑ Mx = 0

10Cy = 5By

By = 2Cy
58

AD = √(30)2 + (15)2

AD = 33,54

AC = √(10)2 + (10)2 + (15)2

AC = 20.62

AB = √(5)2 + (10)2 + (15)2


AB = 18.71

Cy + By = 2000 + Dy

∑ Mz = 0

20Dy = 2000(10)

Dy = 1000

Cy + By = 2000 + 1000

Cy + By = 3000

Cy + 2Cy = 3000

Cy = 1000

By = 2(1000)

By = 2000

2000 FAB
=
15 18.71

FAB = 2495 N
59

② Force of member AC
Cy FAC
=
15 20.62
1000 FAC
=
15 20.62
FAC = 1375 N

③ Force of member AD
Dy FAD
=
15 33.54
1000 FAD
=
15 33.54
FAD = 2236N
60

Chapter

VI
F Riction

- the contact resistance exerted by one body upon a second body when the second
moves or tends to move passing the second body.
- When two objects are in contact, the forces of action and reaction between
contacting surfaces are developed. To the mutual contacting surface, these forces
have their components both in the tangential and normal directions. Force
component in the tangential direction is known as the friction force. Whenever a
tendency exists for one contacting surface to slide along another surface, the
developed friction force is always in the direction opposing this tendency.
- In some systems, friction is undesirable because it just plainly changes the system
characteristics from the required behaviour. In particular, where the sliding motion
between parts occurs, the developed friction force result in a loss of energy.
However, in many cases, friction instead functions the system. For example, many
mechanisms employ friction as an extra force to retain their equilibrium states.
- In short friction is the contact resistance exerted by one body upon a second body
when the second moves or tends to move passing the second body.

F = µN = Frictional force or resisting force

tan Ф = µ

µ = coefficient of friction

F
tan Ф =
N
61

Case I – motion along a straight path


W

R
N
F Ф

Ф F
R
N

Case II – motion
Q along a straight path with multiple forces
P

W
𝛼 𝜃

F R
N
Ф
R
N Ф
F
62

Case III Going upward an inclined plane

F
P

R
𝛼
𝛼 ┌ Ф

Case IV Object going downward an inclined plane


63

Problem: The 200KN block shown has impending motion up that cause the
horizontal force of 400KN. Determine the coefficient of static friction between the
contact surfaces. α = 30°

W = 200KN
W = 200KN
P = 400KN 𝛼

F 𝛼
P = 400KN
F

R
𝛼
𝛼 ┌ Ф
N
N

Solution:

∑ Fv = 0 ↑ +

0 = −400 sin 30° − 200 cos 30° + N


N = 373.21

∑ Fh = 0 → +

0 = 400 cos 30° − 200 sin 30° − F


F = 246.41

F = μN

F
μ=
N

μ = 0.66
64

Problem: The block shown are connected by flexible, inextensible cords passing
over a frictionless pulleys. The coefficient of friction are FA = 0.20andFB = 0.30.
Compute the magnitude and the direction of the frictional force acting on each block.

Assumption: It will slide down at A first

300kN

36.87°

F 𝑇1

∑ Fv = 0 ↑ +

N = 300 cos 36.87°

N = 240

F = μN
65

F = (0.20)(240)

F = 48kN

∑ Fh = 0 → +

0 = −300sin 36.87° + F + T1

T1 = 132kN

200KN

53.13°

𝑇2 F

if body B gowns down... for analysis purposes

∑ Fv = 0 ↑ +

N = 200 cos 53.13°

N = 120 kN

F = μN

F = (0.30)(120)

F = 36kN
66

∑ Fh = 0 → +

0 = −T2 − F + 200sin 53.13°

T2 = 124KN

Evaluation: if 2T1 is greater than T2, body B moves up..

𝑻𝟏
𝑻𝟐

𝑻𝟏

𝟐𝑻𝟏 = 𝑻𝟐
2(132) > (124), therefore, B slides up.

Solution: when B moves up, load W x is still downward

200 kN

53.13°

𝑇2 F

∑ Fh = 0 → +

0 = 200sin 53.13° + F − T2

T2 = 196KN
67

Problem: A horizontal bar 10m long and of negligible weight, rests on rough inclined
planes as shown. If the angle of friction is15°, how close to B may the 200KN force
be applied before motion impends?

2m 100KN 200KN
x =?

F A B

𝟑𝟎° 𝟔𝟎°
10m
Ф = 𝟏𝟓°
𝟑𝟎°
N
𝑹𝑨 𝟒𝟓° Ф = 𝟏𝟓°
F 𝑹𝑩
N

RA
sin 45° =
300
RB RA
R A = 212.13KN
45° 45°

RB 300
sin 45° =
300

R B = 212.13 kN

R Ay = 212.13 × cos 45°

R Ay = 150KN R Ax

R Ax = 212.13 × sin 45° RA


R Ay
R Ax = 150KN
68

R By = 212.13 × cos 45°


R Bx
R By = 150KN

R Bx = 150KN RB

R By

100KN 200KN

2m x =?
A B R Bx
R Ax
10m

R Ay R By

Solve for x:

∑ Mz = 0

0 = −10R By + 100(2) + 200(10 − x)

0 = −10(150) + 100(2) + 2000 − 200x


x = 3.5m
if x > 3.5𝑚, 𝑚𝑜𝑡𝑖𝑜𝑛 𝑖𝑚𝑝𝑒𝑛𝑑𝑠
69

Wedges
Problem: In the figure shown. Determine the minimum height of block B that will keep the
system at rest while a force P starts block A up the inclined surface of B. the weight of A is
100 kN and the angle of friction for all surfaces of contact is15°.

100 KN

P P
A
F
100KN
𝑹𝟏
𝟑𝟎° 𝟒𝟓°
𝟑𝟎°
𝟏𝟓° R
N 100KN

𝑹𝟏

𝟒𝟓°
P

𝒘𝑩

𝟒𝟓° 𝑹𝟏

Ф 𝑹𝟐
N
70

∑ Fy = 0 ↑ +

100
cos 45° =
R1
cos 45°R1 = 100
R1 = 141.42 kN
P
tan 45° =
100
P = 100 kN

sin 15° sin 30°


=
R1 WB 𝟑𝟎°

sin 15° sin 30° 𝑹𝟐 𝟏𝟓° 𝒘𝑩


=
141.42 WB
WB = 273.20 kN
sin 135° sin 15°
=
R2 R1
sin 135° sin 15°
=
R2 141.42
R 2 = 386.37KN

N N
cos 15° = =
R 2 386.37 N 𝟏𝟓° 𝑹𝟐
N = 373.20 kN
F = μN

F = 0.27(373.20)
F = 100.76 kN
71

Belt Friction

T1
= efβ
T2 𝑻𝟏
𝜷
f = coefficient of Friction

β = angle in contact with pulley and


T1 & T2 (expressed in radian)
𝑻𝟐
360 = 2π
72

Problem: A rope is looped over the two fixed posts each of 12cm diameter as shown. If =
1
, determine the maximum and the minimum value of P that will prevent motion of the
π
load W = 1000KN.

240°
max min

T2
T1

T1 T2

max max min


𝟑𝟎°
min 𝟑𝟎°

P
1000KN

Minimum

T1

𝟑𝟎°

1000KN

1000 1 2π
= eπ(30 × 360)
T1
T1 = 846.48 kN
73

𝟐𝟒𝟎°

𝑻𝟐 𝑻𝟏

846.48 1 2π
(240 × 360)
=eπ
T2
T2 = 223.13KN

T2
`
𝟑𝟎°

223.13 1 2π
= eπ(30 × 360)
P
P = 189KN

Maximum

T1

𝟑𝟎°

1000KN

T1 1 2π
= eπ(30 × 360)
1000
T1 = 1181.36KN
74

𝟐𝟒𝟎°

T2 T1

T2 1 2π
= eπ(240 × 360)
1181.36
T2 = 4481.69KN

𝑻𝟐

𝟑𝟎°

P
P 1
(30 ×

)
=e π 360
4481.69
P = 529.49KN
75

Problem: A differential band brake is used to measure the torque output of an engine. The
dimension are shown below. Determine the torque M on the brake when P = 10lb. Assume
the coefficient of kinetic friction to be 0.20.

20” 2”
A T2

T1 ½”
10”
P M
10”

𝜷 = 𝟐𝟕𝟎°

T1
= efβ
T2
76

T2
T1
20”
2”
A
T2
M
½”

P = 10lb T1
β = 270°

T1 2π
= e0.20(270 × 360)
T2

T1
= 2.57
T2

T1 = 2.57 T2 ①

∑ MA = 0

1
0 = −10(20) − T1 ( ) + T2 (2)
2

1
0 = −200 − T1 ( ) + 2T2
2

Subs ① in ②
T2
0 = −200 − ( ) (2.57) + 2T2
2
T2 = 279 lb
T1 = 717 lb
77

∑ MC = MC

M = T1 (10) − T2 (10)
M = 717(10) − 279(10)
M = 4380 lb
78

Friction
Problem: The motorcyclist travels with constant velocity along a straight, horizontal,
banked road. If he aligns his bike so that the tires are perpendicular to the road at A,
determine the frictional force at A. The man has a mass MC and a mass center at GC, and
the motorcycle has a mass Mm and a mass center at Gm. If the coefficient of static friction
at A is μA, will the bike slip?

Given:
MC = 60 kg
Mm = 120 kg
μA= 0.4
θ = 20 deg

m
g = 9.81
s2
Solution: Assume no slipping
ΣFy= 0; NA − (Mm + Mc) g cos (θ) = 0
NA = (Mm + Mc) g cos (θ)
NA = 1659N

ΣFx= 0; FA − (Mm + Mc) g sin (θ) θ


FA = (Mm + Mc) g sin(θ)
FA = 604N
FAmax= μA NA

Check: If FA = 604 N <FAmax= 664 N


then our no-slip assumption is good
79

Problem: The crate has a mass M and is subjected to a towing force P acting at an angle
θ1with the horizontal. If the coefficient of static friction isμ s , determine the magnitude of P
to just start the crate moving down the plane.
Given:
M = 350 kg
θ1 = 20 deg P
θ 2 = 10 deg
μ s = 0.5 θ1
m
g = 9.81 2
s
θ2

Solution:
Given
Initial guesses
P = 20N
N=10N

ΣF x = 0;
P cos (θ1 + θ 2 )− μ s N + M g sin(θ 2 )= 0

ΣF y = 0;

N − M g cos (θ 2 )+ P sin(θ1 + θ 2 )= 0

Find (N, P)
N = 2891 N
P = 981
80

Problem: The coefficients of static friction between the 100kg block and the inclined plane
is 0.30. Determine (a) the friction force F acting on the block when P is applied with a
magnitude of 200N to the block at rest, (b) the force P required to initiate motion up the
incline from rest, and (c) the friction force F acting on the block if P = 600N.

P
100 kg
20°

15°

(a)

∑ Fv = 0 ↑ +

N = −200 sin 20° + 981 cos 15°

N = 879.169

F = μN

F = (0.30)(879.169)

F = 263.75

(b)

∑ Fv = 0 ↑ +

N = −P sin 20° + 981 cos 15° ①


81

∑ Fh = 0 ↑ +

0 = −F + P cos 20° −981 sin 15° ②

① in ②

0 = −[(0.30)(918 cos 15° − P sin 20°)] + P cos 20° −981 sin 15°
918 cos 15°(0.30) − 981 sin 15°
P=
sin 20°(0.30) + cos 20°
P = 516.33

(c)

∑ Fv = 0 ↑ +

N = −600 sin 20° + 981 cos 15°

N = 742.36

F = μN

F = (0.30)(742.36)

F = 222.708
82

Problem: A block weighing 500N just starts moving down a rough inclined plane when
supported by a force of 200N acting parallel to the plane in upward direction. The same
block is on the verge of moving up the plane when pulled by a force of 300N acting parallel
to the plane. Find the inclination of the plane and coefficient of friction between the inclined
plane and the block.
500 kN
∑ Fv = 0 200 kN
N = 500 cos θ ①

F1 = μN = μ(500 cos θ) 𝐹1

∑ Fh = 0 𝜃 N

200 + F1 = 500 sin θ

500 kN

∑ Fv = 0 ②
300 kN
N = 500 cos θ
𝐹2
F2 = μN = μ(500 cos θ)
N
𝜃

∑ Fh = 0

500 sin θ + F2 = 300


500 sin θ + μ(500 cos θ) = 300

Adding ② and①
We get,
500 = 1000 sin θ
sin θ = 0.5
θ = 30°
83

Subs. θ in②
500 sin 30° + μ(500 cos 30°) = 300

50
μ= = 0.11547
500 cos 30°
84

Problem: The light bar is used to support the 50 kg block in its vertical guides. If the
coefficient of static friction is 0.30 at the upper end of the bar and 0.40 at the lower end of
the bar, find the frictional force acting at each end for x = 75mm. also find the maximum
value of x for which the bar will not slip.

x = 75 mm
75
θ = sin−1 = 14.5°
300

ФA = tan−1 (0.40) = 21.8°

B
ФB = tan−1 (0.30) = 16.7°
30
0
FA = FB = W tan θ m
FA = 50(9.81) tan 14.5° m
A
FA = 126.6 N x

x = 300 sin 16.7°


x = 86.2 mm
85

Chapter

VII
C entroid

– is the central point in a length, an area and a volume.


Center of Gravity– is the central point for the distribution of mass and gravitational force.

SHAPE AREA OR LENGTH x̅ y̅

bd b d
2 2

1 2 1
bd b d
2 3 3

πr 2 x=r 4r
2 3π
86

1 2 4r 4r
πr
4 3π 3π

r 2α 2 sinα 0
r
3 α

2rα rsinα 0
α
87

Problem: Find the center of gravity with respect to point A.

Sol.
Point A
4(3)
A=− = 6m2
2
2 2
x̅ = 3 b = 3 (3) = 2m
1 1 4
y̅ = d = (4) = m
3 3 3

Point B
1 1
x̅ = b = (3) = 1m
3 3
2 2 8
y̅ = b = (4) = m
3 3 3
88

Problem: Locate the value of x & y with respect to point A.

Sol.

πr 2 (4)2
A1 = =π = 12.57m2
4 4

A2 = bd = 3(4) = 12m2

1 1
A3 = bd = (4)(3) = 6m2
2 2

ATOTAL = A1 + A2 + A3 = 30.57m2

4r
x1 = 4 − = 2.30m
3h
89

b
x2 = 4 + = 5.50m
2
,
2
x3 = 4 + b = 6.00m
3

ATOTAL x̅ = A1 X1 + A2 X2 + A3 X3
30.57x̅ = 12.57(2.30) + 12(5.50) + 6(6.00)
x̅ = 4.28m

4r
,y1 = 3 h = 1.70m
d
,y2 = 2 = 2.00m
1
,y3 = 4 + 3 d = 5.33m

ATOTAL y̅ = A1 Y1 + A2 Y2 + A3 Y3

30.57y̅ = 12.57(1.70) + 12(2.00) + 6(5.33)


y̅ = 2.53
90

Problem: Find x & y with respect to point A.

1 1
A1 = bd = (3)(7) = 10.5m2
2 2

A2 = bd = (5)(7) = 35m2

πr 2 π(3)2
A3 = = = 7.07m2
4 2

ATOTAL = A1 + A2 − A3 = 38.43m2

1
x1 = 5 + b = 6.00m
3

1
,x2 = 2 b = 2.50m

4r
x3 = = 1.27m
3h

ATOTAL = A1 X1 + A2 X2 − A3 X3
38.43x̅ = 10.5(6.00) + 35(2.50) − 7.07(1.27)
91

x̅ = 3.68m

1
y1 = d = 2.33m
3

1
y2 = d = 3.50m
2

4r
y3 = = 1.27m
3h

ATOTAL y̅ = A1 Y1 + A2 Y2 − A3 Y3
38.43y̅ = 10.5(2.33) + 35(3.50) + 7.0(1.27)
y̅ = 3.59m
92

Problem: Determine the total area of the given figure.

Sol.
A1 = bd = (20 + 30)(20) = 2000cm2

πr 2 π(20)2
A2 = = = 628.32cm2
2 2

A3 = πr 2 = π(10)2 = 314.16cm2

ATOTAL = A1 + A2 − A3 = 2314.16cm2
93

Problem: The centroid of the shaded area in the figure shown is required to lie in the y-
axis. Determine the distance “b” that will fulfil this requirement.

Sol:
A1 = 4(8) = 32

A2 = 6b

A = A1 + A2
Ax̅ = A1 X1 + A2 X 2
2 2
A(0) = 32 ( ) (8) + 6b ( b )
3 3
32(8) = 6b2
b = 6.54m
94

Problem: From given section below:

Sol.
For area of the shaded section

π(4)2
A = 10(2) + 6(4) −
4
A = 20 + 24 − 12.566
A = 31.43m2
95

Centroid from y-axis


Ax̅ = A1 Y1 + A2 Y2 − A3 Y3
31.43x̅ = 20(1) + 24(4) − 12.566(4.3)
y̅ = 1.97m

Centroid from x-axis


Ax̅ = A1 X1 + A2 X 2 − A3 X3
31.43x̅ = 20(5) + 24(3) − 12.566(4.3)
x̅ = 3.75m
96

Problem: A slender homogeneous wire of uniform cross sectionis bent into a shape
shown. Determine the coordinates of its centroids.

Sol.
L = L1 + L2 + L3
L1 = 6
L2 = π(4)
L3 = 8

L = 26.57
Lx̅ = L1 X1 + L2 X 2 + L3 X3
26.57x̅ = 6(−4) + 4π(0) + 8(4 + 4 cos 30o )
x̅ = 1.34m

Ly̅ = L1 Y1 + L2 Y2 + L3 Y3
2(4)
26.57y̅ = 6(3) + 4π ( ) + 8(4 sin 30o )
π

y̅ = 2.48m
97

Problem: Determine its area and its centroid coordinate.

Sol.
A = A1 − A2 − A3
π(1)2
A = 8(4) − 2(2) −
2
A = 26.43sq. m.

X-coordinate of centroid meters

Sol.
Ax = A1 X1 − A2 X 2 −A3 X3
π
26.43x = 32(0) − 4(−4) − (1)2 (2)
2
x = +0.18
98

Chapter

VIII

M
Transfer formula
oment of Inertia

: Ix = Ixo + Ady2

: Iy = Iyo + Adx2

Shape Moment of Inertia

bh3 b3 h
Ixo = ;I =
12 yo 12

bh3 b3 h
Ix = ; Iyo =
3 3

103
99

bh3 b3 h
Ixo = ;I =
36 yo 36

bh3 b3 h
Ix = ; Iyo =
12 12

πr 4 πr 4
Ixo = ; Iyo =
4 4

πr 4
J=
2

πr 2
Ix. = Iyo =
8

Ixo = 0.11r 4

Ixo = Iyo = 0.55r4

πr 4
Ix = Iy =
16
100

Problem: Determine the amount of inertia for a rectangle in the figure shown. At so, if it is
measured with respect to point A.

bh3 (3)(4)3
Ixo = =
3 3
Ixo = 16m4

b3 h (3)3 (4)
Iy′o = =
12 12
Iyo = 9m4

A = bh = 3(4) = 12m2

IxA = Ixo + Ady 2


IxA = 16 + 12(2)2
101

IxA = 64m4

IyA = Iyo + Ady 2

IyA = 9 + 12(0.5)2

Iyh = 12m4
102

Problem: Compute the moment of inertia with respect to Xo &Yo

Sol.
A1 = bh = (3.5)(10) = 35m2
A2 = bh = (8)(12) = 96m2

bh3 (3.5)(10)2
IxoS = x12 = x12
12 12
IxoS = 583.33m4

bh3 (8)(12)3
IxoB = =
12 12
IxoB = 1152m4

Ixo = IyoB − IyoS

Ixo = 1152 − 583.33


Ixo = 568.67m4

b3 h (8)3 (12)
IyoB = =
12 12
103

IyoB = 512m4

IyS = IyoS + Adx 2

b3 h (3.5)3 (10)
IyoS = =
12 12
IyoS = 35.73m4

IyS = 35.73 + (35)(2.25)2

IyS = 212.92m4

Iyo = IyoB − 2 x IyS

Iyo = 5 − 12 − 2 x 212.92

Iyo = 86.16m4

What is the IyB with respect to point B?

IyoB = 512m4

IyBB = IyoB + ABdyb2

IyBB = 512 + 96(2.25)2

IyBB = 998m4

IyBs1 = Iyos1 + As1 dxs12

IyBs1 = 35.73 + 36(45)2

IyBs1 = 744.48m4

IBs2 = Iyos2 + As1 dxs22

IBs2 = 35.73m4
104

IyB = IyBB − IyBs1 − IyBs2

IyB = 998 − 744.48 − 35.73

IyB = 217.79m4
105

Problem: Compute the product of inertia of the triangular area shown in the figure with
respect to centroidal axes parallel to the given x and y axes.

Sol.
̅xy + Ax̅y̅
Pxy = P
1 1
̅xy + (3)(9)(2)(3) + (6)(9)(5)(3)
455.625 = P
2 2
̅xy = 30.375m4
P
106

Problem: For a certain, it is known that lx = 60in4 , ly = 20in4 andPxy = 0. Find the moment
of inertia of this area with respect to U-axis qhich is rotated 30o counterclockwise from the
x-axis.

Sol.
2R + 20 = 60
R = 20

Lay out an angle double that of 30o (that is 60o ) counterclockwise from the horizontal
axes.
lU = 20 + R + R cos 30o
lU = 20 + 20 + 20 cos 30o
lU = 50in4
107

Problem: Determine the moment of inertia of the T-section with respect to its centroidal xo
axis

Sol:
A1 = 2(8) = 16

A2 = 2(8) = 16

A = A1 + A2
A = 32

Ay̅ = A1 y1 + A2 y2

32y̅ = 16(1) + 16(6)


y̅ = 3.5

2(8 )3 8(2)3
lxo = + 2(8)(2.5)2 + + 8(2)(2.5)2
3 12
lxo = 290.67m4
108

Problem: A hollow square cross section consists of an 8m x 8m square from which is


subtracted a concentrically place square 4m x 4m. Find the polar moment of inertia and
the polar radius of gyration with respect to z axis passing through one of the outside
corners.

Sol.
8(8) 3 4(4)3
lx = −[ + 4(4)(4)2 ]
3 12

lx = 1088m4
ly = 1088m4

J = lx + ly

J = 1088 + 1088
J = 2176

2176
k=√
(8)2 −(4)2

k = 6.73
109

Problem: A rectangle is 3in x 6in. Determine the polar moment of inertia and the radius of
gyration with respect to a polar axis through one corner.

Sol.
(3)(6)3
lx = = 216
3
6(3) 3
ly = = 54
3

J = lx + ly

J = 216 + 54
J = 270in4

270
k=√
3(6)

k = 3.87
110

References

Books

Engineering Mechanics (Statics and Dynamics) Second Edition, 2009, Besavilla Jr.
Venancio I., ISBN 971-8510-04-4.

Engineering Mechanics (Statics) Fifth Edition, 2002, Meriam, J. L. and Kraige, L.G.,
ISBN 0-471-40646-5, by John Wiley and Sons, Inc.

Learning Guide in Engineering Mechanics, 1996, Dela Rama, Nicanor C. and


Mendoza, Alfredo G., ISBN 971-23-1997-0, by Rex Bookstore.

Internet Website

https://www.mathalino.com

Das könnte Ihnen auch gefallen